Đến nội dung

viet9a14124869 nội dung

Có 855 mục bởi viet9a14124869 (Tìm giới hạn từ 29-04-2020)



Sắp theo                Sắp xếp  

#671991 Chuyên đề Hệ phương trình

Đã gửi bởi viet9a14124869 on 18-02-2017 - 17:19 trong Chuyên đề toán THPT

bài này khi nhân chéo rồi thực hiện bình phương thì nhận thấy cần cm $c(a-b)(c^2-ab)\geq 0\Leftrightarrow$ $a\geq b$ nên chắc thiếu điều kiện  

mọi người giúp mình câu này ạ: 

cho a,b,c thỏa mãn a,b>0 và c>$\sqrt{ab}$ . chứng minh rằng:

$\frac{a+c}{\sqrt{a^2+c^2}} \geq \frac{b+c}{\sqrt{b^2+c^2}}$

 

giải hệ phương trình:

a, $\left\{\begin{matrix} xy(x^2+y^2)=2 & \\ 2x^5=(x+y)(x^4+y^4+x^2y^2-2) & \end{matrix}\right.$

b, $\left\{\begin{matrix} \sqrt[3]{x+2y}=4-x-y & \\ \sqrt[3]{2x+6} +\sqrt{2y} =2 & \end{matrix}\right.$

c, $\left\{\begin{matrix} x^2(1+y^2)=2 & \\ 1+x^2y+xy=3x^2 & \end{matrix}\right.$

d, $\left\{\begin{matrix} y^2+4xy+y-2x=0 & \\ y^4+8xy^2+4x^2+3y^2=0 & \end{matrix}\right.$

e, $\left\{\begin{matrix} 4x-4y^2=x^2y^2 & \\ 3x^2+y^3=12x-13 & \end{matrix}\right.$

mọi người giúp em với ạ

a, thay $xy(x^2+y^2)=2$ vào phương trình 2 thì được $2x^5=x^5+y^5\Rightarrow x=y$ ...đến đây dễ suy ra nghiệm .

e, Từ phương trình 1 suy ra $y^2=\frac{4x}{x^2+4}\leq 1\Rightarrow -1\leq y\leq 1$

Mà theo phương trình 2 thì $y^3+1=3(x-2)^2\geq 0\Rightarrow y\geq -1$

Do đó y=-1,,,x=2




#674047 Topic về Bất đẳng thức, cực trị THCS

Đã gửi bởi viet9a14124869 on 12-03-2017 - 13:18 trong Bất đẳng thức và cực trị

Cho x,y thỏa mãn

$\left ( x^{2}+y^{2}+1 \right )^{2}+4x^{2}y^{2}-x^{2}-y^{2}=0$

Tìm min, max P=$x^{2}+y^{2}$

Đề sai bạn ạ ,,,khi tách hết ra ta được $x^4+6x^2y^2+y^4+x^2+y^2+1=0$ vô lí




#672777 Topic về Bất đẳng thức, cực trị THCS

Đã gửi bởi viet9a14124869 on 25-02-2017 - 22:21 trong Bất đẳng thức và cực trị

Cho a,b,c>0 thỏa $a+b+c\geq 3$.Tìm giá trị nhỏ nhất của biểu thức S=$\frac{a}{\sqrt{b}}+\frac{b}{\sqrt{c}}+\frac{c}{\sqrt{a}}$

Cho x,y,z>0 thỏa mãn x+y+z=1. Tìm giá trị lớn nhất của biểu thức: P=$\frac{x}{x+1}+\frac{y}{y+1}+\frac{z}{z+1}$

1, Ta có $\frac{a}{\sqrt{b}}+\frac{b}{\sqrt{c}}+\frac{c}{\sqrt{a}}=\frac{a^2}{a\sqrt{b}}+\frac{b^2}{b\sqrt{c}}+\frac{c^2}{c\sqrt{a}}\geq \frac{(a+b+c)^2}{a\sqrt{b}+b\sqrt{c}+c\sqrt{a}}\geq \frac{(a+b+c)^2}{\sqrt{(a+b+c)(ab+bc+ca)}}=\frac{9}{\sqrt{3(ab+bc+ca)}}\geq 3\Leftrightarrow a=b=c=1$

2, Bạn thay x+y+z=1 vào mẫu mỗi phân số rồi dùng cô-si là xong




#675998 Topic về Bất đẳng thức, cực trị THCS

Đã gửi bởi viet9a14124869 on 02-04-2017 - 15:55 trong Bất đẳng thức và cực trị

Cho a,b,c là các số thực dương thỏa mãn abc=1.Chứng minh:

$\frac{a}{(a+1)(b+1)}+\frac{b}{(b+1)(c+1)}+\frac{c}{(c+1)(a+1)}\geq \frac{3}{4}$

Cho ba số dương a,b,c thỏa $a+b+c\leq k$ thì $(1+\frac{1}{a})(1+\frac{1}{b})(1+\frac{1}{c})\geq (1+\frac{3}{k})^3$

Câu đầu đồng quy biểu thức ,,,ta cần chứng minh $ab+bc+ca+a+b+c\geq 6$ đúng theo AM-GM

Câu sau ta dùng holder với AM-GM ta có $\prod (\frac{1}{2}+\frac{1}{2}+\frac{1}{a})\geq (1+\frac{1}{\sqrt[3]{abc}})^3\geq (1+\frac{3}{a+b+c})^3\geq (1+\frac{3}{k})^3$




#676656 Topic về Bất đẳng thức, cực trị THCS

Đã gửi bởi viet9a14124869 on 08-04-2017 - 21:12 trong Bất đẳng thức và cực trị

Cho a,b,c>0 và a^2+b^2+c^2=1.CM $\frac{bc}{a^2+1}+\frac{ca}{b^2+1}+\frac{ab}{c^2+1}\leqslant \frac{3}{4}$

Ta có $\frac{b^2}{a^2+b^2}+\frac{c^2}{c^2+a^2}\geq \frac{(b+c)^2}{2a^2+b^2+c^2}\geq \frac{4bc}{2a^2+b^2+c^2}=\frac{4bc}{a^2+1}$

Làm tương tự rồi cộng 3 vế lại ta có đpcm !

Dấu = xảy ra khi $a=b=c=\frac{1}{\sqrt{3}}$




#677413 Topic về Bất đẳng thức, cực trị THCS

Đã gửi bởi viet9a14124869 on 14-04-2017 - 20:45 trong Bất đẳng thức và cực trị

Nếu sử dụng $AM-GM$ dưới mẫu sẽ không thỏa mãn dấu bằng vì $a=b=c=3$ mà.

Bạn tách ra $a+bc=a+\frac{bc}{3}+\frac{bc}{3}+\frac{bc}{3}$ rồi dùng AM-GM  là OK




#677033 Topic về Bất đẳng thức, cực trị THCS

Đã gửi bởi viet9a14124869 on 11-04-2017 - 06:06 trong Bất đẳng thức và cực trị

cho các số thực $a,b,c$. Chứng minh :

$$\frac{ab}{a+b-c}+\frac{bc}{b+c-a}+\frac{ca}{c+a-b}\ge a+b+c$$

 

Đề ra cho là sai $a,b,c\ge 0$ mới đúng

Mình nghĩ nên có thêm điều kiện a,b,c là 3 cạnh tam giác sẽ đúng hơn ,,bởi nếu a=1,b=2,c=4 thì bất đẳng thức sai




#677410 Topic về Bất đẳng thức, cực trị THCS

Đã gửi bởi viet9a14124869 on 14-04-2017 - 20:35 trong Bất đẳng thức và cực trị

1. Cho 3 số  dưeơng a,b,c. CMR: $(\frac{a}{b}+\frac{b}{c}+\frac{c}{a})^{2}\geq(a+b+c)(\frac{1}{a}+\frac{1}{b}+\frac{1}{c})$

2. Cho các só thực x,y, z thỏa mãn điều kiện: x2 +y+z2=1. Tìm GTLN của A=xy+yz+2xz

3. Cho các số thực a,b,c thuộc đoạn [-2;5] tm: a+2b+3c$\leq$2. Tìm GTLN: a2+2b2+3c2

4. Cho a,b,c>0 tm: $\frac{1}{a}+\frac{1}{b}+\frac{1}{c}$=1. CMR:$\frac{a^{2}}{a+bc}+\frac{b^{2}}{b+ca}+\frac{c^{2}}{c+ab}\geq \frac{a+b+c}{4}$

Bài 2 , Ta có $\left\{\begin{matrix} xy\leq \frac{\sqrt{3}-1}{2}x^2+\frac{1}{2\sqrt{3}-2}y^2 & & \\ yz\leq \frac{\sqrt{3}-1}{2}z^2+\frac{1}{2\sqrt{3}-2}y^2 & & \\ 2xz\leq x^2+z^2 & & \end{matrix}\right.\Rightarrow xy+yz+2zx\leq \frac{\sqrt{3}+1}{2}(x^2+y^2+z^2)=\frac{\sqrt{3}+1}{2}\Leftrightarrow x=z=\frac{\sqrt{3}+1}{2}y\Leftrightarrow .........$

Bài 3 , $-2\leq a\leq 5\Leftrightarrow a^2\leq 3a+10 \Rightarrow ..... \Rightarrow a^2+2b^2+3c^2\leq (3a+10)+2(3b+10)+3(3c+10)=3(a+2b+3c)+60\leq  66\Leftrightarrow a=-2,b=5,c=-2$

Bài 4 , Chắc là dùng $\frac{a^2}{a+bc}=a-\frac{abc}{a+bc}$ rồi dùng AM-GM ở mẫu ^_^




#673447 Hướng dẫn gửi bài trên Diễn đàn

Đã gửi bởi viet9a14124869 on 04-03-2017 - 21:18 trong Hướng dẫn - Trợ giúp - Giải đáp thắc mắc khi sử dụng Diễn đàn

 

không thấy gửi bài mới ở đâu

 

https://diendantoanh...-và-cực-trị/

Giả dụ bạn muốn gửi bài trong box Bất đẳng thức thì bấm vào link trên này ,,,sẽ thấy có ô màu đen ghi gửi bài mới đó




#671833 Topic phương trình, hệ phương trình vô tỉ

Đã gửi bởi viet9a14124869 on 16-02-2017 - 21:23 trong Phương trình, hệ phương trình và bất phương trình

\[ Bài 1 : {x^3} + 2\sqrt {{{(3x - 2)}^3}}  = 3x(3x - 2)\]

 Đặt $\sqrt{3x-2}=a\Rightarrow x^3+b^3+b^3=3xb^2\Leftrightarrow x+2b=0$ hoặc $x^2-2bx+3b^2=0$ loại

Vậy ta sẽ tìm được x do x+2b=0




#671851 Topic phương trình, hệ phương trình vô tỉ

Đã gửi bởi viet9a14124869 on 17-02-2017 - 06:11 trong Phương trình, hệ phương trình và bất phương trình

Sai ! thử thay số vào đi bạn 

Nếu thay số mà không  ra thì chắc là vô nghiệm




#671625 Topic phương trình, hệ phương trình vô tỉ

Đã gửi bởi viet9a14124869 on 14-02-2017 - 19:51 trong Phương trình, hệ phương trình và bất phương trình

\[ Bài 3 : {x^2} = \sqrt {{x^3} - {x^2}}  + \sqrt {{x^2} - x} \]

Xét thấy x=0 là nghiệm của bài toán

Nếu $x\neq 0\Rightarrow x\geq 1\Rightarrow 2x^2=2\sqrt{x^2(x-1)}+2\sqrt{(x^2-x).1}\leq (x^2+x-1)+(x^2-x+1)=2x^2\Leftrightarrow x^2=x-1=1-x\Leftrightarrow 1=x=0$ loại

Vậy x=0




#671787 Topic phương trình, hệ phương trình vô tỉ

Đã gửi bởi viet9a14124869 on 16-02-2017 - 13:35 trong Phương trình, hệ phương trình và bất phương trình

máy mình cũng bị tương tự

giải phương trình 4{x-\sqrt(5-x)}.{\sqrt(5-x) +3}=(x+3)^2

 

bạn nào sửa hộ mih đk máy mih gặp sự cố

Sử dụng bdt cauchy cho 2 số ta có 4ab<=(a+b)^2 ....

Do đó ta cm được VT<=VP 




#671623 Topic phương trình, hệ phương trình vô tỉ

Đã gửi bởi viet9a14124869 on 14-02-2017 - 19:46 trong Phương trình, hệ phương trình và bất phương trình

\[ Bài 2 : \sqrt {4{x^2} + 5x + 1}  + 3 = 2\sqrt {{x^2} - x + 1}  + 9x\]

Ta có $\Leftrightarrow \sqrt{4x^2+5x+1}-\sqrt{4x^2-4x+4}=9x-3\Leftrightarrow \frac{9x-3}{\sqrt{4x^2+5x+1}+\sqrt{4x^2-4x+4}}=9x-3$

Xét $x\neq \frac{1}{3}\Rightarrow 1=\sqrt{4x^2+5x+1}+\sqrt{4x^2-4x+4}\geq \sqrt{3}> 1$ loại

Do đó $x=\frac{1}{3}$




#671667 Topic: Các bài toán về tính chia hết

Đã gửi bởi viet9a14124869 on 14-02-2017 - 22:30 trong Số học

Số dư khi chia $[n^{2}+1]^{2016} cho n$ là bao nhiêu

1 theo đồng dư thức ^-^




#676171 Topic: Các bài toán về tính chia hết

Đã gửi bởi viet9a14124869 on 04-04-2017 - 12:40 trong Số học

- Chứng minh rằng 210+512 là hợp số .
Mod và các bác giúp em với :((
Em cần gấp :3

Đề sai ,,vì $2^{10}+5^{12}=244141649$ là số nguyên tố :3




#669532 MỘT SỐ PHƯƠNG PHÁP GIẢI TOÁN TỔ HỢP THCS

Đã gửi bởi viet9a14124869 on 23-01-2017 - 12:40 trong Toán rời rạc

 

Đếm số hình tam giác:

attachicon.gifunnamed.png

có 32 tam giác

 

 

bài hay đây:

với x, y, z là các số dương CM

x^2(x+y-z)+y^2(y+z-x)+z^2(x+y-z)bé hơn hoặc bằng 3xyz

 

 

bài này bạn áp dụng bdt hoán vị cho 2 bộ đơn điệu sau $(x,y,z)$ và $(x^2,y^2,z^2)$

 




#669907 MỘT SỐ PHƯƠNG PHÁP GIẢI TOÁN TỔ HỢP THCS

Đã gửi bởi viet9a14124869 on 25-01-2017 - 21:10 trong Toán rời rạc

Bài 23: Cho $a_{1},a_{2},a_{3},................a_{n}$ là 1 hoán vị của 1, 2, 3,............n với n là số lẻ. CMR : $\left ( a_{1}-1 \right )\left ( a_{2} -2\right ).............\left ( a_{n} -n\right )$ là số chẵn . Cho em hỏi hoán vị nghĩa là sao ạ?

nghĩa là các số từ a1 đến an là các số từ 1 đến n nhưng trât tự thay đổi

Vì $(a_{1}-1)+(a_{2}-2)+...+(a_{n}-n)=0$ nên tồn tại ít nhất một hiệu chẵn suy ra q.e.d




#671450 MỘT SỐ PHƯƠNG PHÁP GIẢI TOÁN TỔ HỢP THCS

Đã gửi bởi viet9a14124869 on 13-02-2017 - 12:52 trong Toán rời rạc

không

mình lại nghĩ là có ,,,nhưng chỉ đoán qua thôi ,,,vì số bóng đèn chia hết cho 2 ^-^




#671824 MỘT SỐ PHƯƠNG PHÁP GIẢI TOÁN TỔ HỢP THCS

Đã gửi bởi viet9a14124869 on 16-02-2017 - 20:40 trong Toán rời rạc

Có thể tìm a;b;c là số nguyên nếu a^3+b^3=c^3? Tại sao thế?
Tớ đóng góp vậy đó, xin được giúp đỡ

Ta chọn a=-b và c=0 là xong




#670739 Diễn đàn đã hoạt động trở lại

Đã gửi bởi viet9a14124869 on 08-02-2017 - 19:49 trong Thông báo tổng quan

vậy có cách nào cải thiện tình huống của bạn canhhoang30011999 ko ,,, em dùng firefox cũng bị như vậy ạ 




#670744 Diễn đàn đã hoạt động trở lại

Đã gửi bởi viet9a14124869 on 08-02-2017 - 20:04 trong Thông báo tổng quan

dạ thôi em vào dc rồi anh ạ ^-^




#670743 Diễn đàn đã hoạt động trở lại

Đã gửi bởi viet9a14124869 on 08-02-2017 - 20:02 trong Thông báo tổng quan

em thử bỏ rồi nhưng khi gõ enter là lại có chữ s nữa anh ạ 




#672093 Diễn đàn đã hoạt động trở lại

Đã gửi bởi viet9a14124869 on 19-02-2017 - 16:15 trong Thông báo tổng quan

Cho em hỏi tại sao máy tính của em ko thể kết nối vs diễn đàn vậy ạ?

thế bạn đăng câu này kiểu gì ,,,,




#680284 Topic ôn thi hình học vào cấp 3 chuyên

Đã gửi bởi viet9a14124869 on 11-05-2017 - 13:38 trong Hình học

                                                                                          :D  " Góp vui cho topic "  :D 

Mình xin tham gia topic và mở đầu bằng hai bài toán .

Bài 89 ( APMO 2000 ): Cho tam giác ABC với trung tuyến AM và phân giác AN . Đường thẳng vuông góc với AN tại N cắt AB và AM lần lượt tại P và Q . Đường thẳng vuông góc với AB tại P cắt AN tại O . Chứng minh rằng OQ vuông góc với BC . 

Bài 90 ( sưu tầm ) : Cho tam giác ABC có I là trung điểm BC , đường thẳng d đi qua I cắt AB , AC lần lượt tại M và N , đường thẳng d' đi qua I cắt AB ,AC lần lượt tại Q và P ( M và P nàm cùng phía với BC ) . MP , NQ cắt BC tại E và F . Chứng minh rằng IE = IF .